Separation of variables and generalised fourier series

Click For Summary
The discussion revolves around solving the equation \nabla^2u = 0 with specific boundary conditions in a defined rectangular domain. The user is attempting to derive the solution u(x,y) = \frac{x}{2\pi} + \frac{cos2ysinh2x}{2sinh2\pi} but is confused about applying separation of variables correctly. They have successfully solved a similar problem but struggle with the current boundary conditions, particularly how they affect the functions X(x) and Y(y). The user questions whether it suffices to verify that the proposed solution meets the equation and boundary conditions without deriving it fully. The conversation highlights the complexities of applying separation of variables in different contexts.
catcherintherye
Messages
47
Reaction score
0

Homework Statement



if \nabla^2u = 0 in 0 \leq x \leq \pi, 0\leq y \leq \pi,

boundary conditions u(0,y)=0, u(\pi,y)=cos^2y, u_y(x,0) = u_y(x,\pi)=0


Homework Equations



I am required to show that u(x,y) = \frac{x}{2\pi} + \frac{cos2ysinh2x}{2sinh2\pi}

The Attempt at a Solution




I have done similar questions before for example consider then following problem:

\nabla^2u = 0 in 0 \leq x \leq a, 0\leq y \leq b,

but with boundary conditions:

u(0,y)=0, u(a,y)=0, u(x,0)=0, u(x,b)=f(x)


I derived the general solution to be u(x,y) = \sum_{n=0}^\infty D_nsin\frac{n\pix}{a}sinh\frac{n\piy}{a}

with D_n =\frac{2}{asinh\frac{n\pib}{a}}\int_{0}^{a}f(x)sin\frac{n\pix}{a}dx n=1,2,3...



...This example I understand but in the first example I am confused so far I have done the following,

let u(x,y) =X(x)Y(y)

X(x) =Acospx + Bsinpx
Y(y) =Ccoshpy + Dsinhpy
then Y'(y) =Esinhpy + Fcoshpy

B.C's ->

X(0) =A=0

X(\pi)= Bsinp\pi=cos^2y


Y&#039;(0) = F=0, Y&#039;(\pi)=0<br /> <br /> so Esinhp\pi=0 this seems to tell me nothing about E I have tried also type three solutions viz.

X(x)=Acoshpx + Bsinhpx

Y(y)= Ccospy + D sinpy
Y'(y)=Ecospy-Fsinpy

X(0)=A=0
X(\pi)= Bsinhp\pi =cos^2(y)

Y'(0)=E=0

Y&#039;(pi)= -Fsinppi=0

when p = 1,2,3

so u(x,y) = \sum_{p=0}^\infty F_nsinpxsinhpy

which is markedly different to the form of solution required, what am I doing wrong??
















 
Last edited:
Physics news on Phys.org
If you only need show that the expression you give is the solution, can you not just show it satisfies the equation and that it satisfies the boundary conditions?
 
Question: A clock's minute hand has length 4 and its hour hand has length 3. What is the distance between the tips at the moment when it is increasing most rapidly?(Putnam Exam Question) Answer: Making assumption that both the hands moves at constant angular velocities, the answer is ## \sqrt{7} .## But don't you think this assumption is somewhat doubtful and wrong?

Similar threads

  • · Replies 11 ·
Replies
11
Views
3K
  • · Replies 7 ·
Replies
7
Views
2K
Replies
3
Views
2K
  • · Replies 10 ·
Replies
10
Views
2K
  • · Replies 3 ·
Replies
3
Views
2K
  • · Replies 14 ·
Replies
14
Views
2K
Replies
4
Views
2K
  • · Replies 8 ·
Replies
8
Views
2K
  • · Replies 6 ·
Replies
6
Views
2K
  • · Replies 2 ·
Replies
2
Views
2K